Lesson Eleven: Logical Reasoning Flashcards

1
Q

–LOGICAL REASONING: Evaluate the Argument–

A
How well did you know this?
1
Not at all
2
3
4
5
Perfectly
2
Q

Evaluate the Argument

(Question stem #, Family, Model, Definition)

A

Question Stem:
#12

Family:
Combination of Families 2 and 3

Model:
Stimulus
↑⍏
Answer Choice

Definition:
Question stem that asks you to choose the answer choice that strengthens and weakens the conclusion; reveals the validity of the argument.

How well did you know this?
1
Not at all
2
3
4
5
Perfectly
3
Q

These question stems are a combination of ____________ and _________ questions. The answer is in the form of a _________, to which the response(s) to that answer choice should ___________ and __________ the weaknesses of the argument’s conclusion.

A

strengthen; weaken; question; reveal; eliminate

How well did you know this?
1
Not at all
2
3
4
5
Perfectly
4
Q

Process of Answering Evaluate Question

A

In the stimulus, you will be presented with an argument.

The question stem will present you with answer choices in the form questions, asking you which one of the following questions would be most useful in evaluating the conclusion of the argument.

In evaluating the conclusion of an argument, we want to know what would strengthen and weaken the conclusion of an argument.

So, for the answer choices (in the form of questions), we are going to give two polar opposite test responses to the question. If each response has a different effect on the conclusion of the argument, such that one response strengthens the argument, and the other weakens the argument, then it is the correct answer choice.

If the each response does not have a different effect on the conclusion of the argument, then it is the incorrect answer choice.

How well did you know this?
1
Not at all
2
3
4
5
Perfectly
5
Q

Because Evaluate questions are a combination of families #2 and #3, keep in mind that…

1) in all question stem types apart of families #2 and #3, the ______________ in the stimulus is _______________, so you should be reading for _____________ in reasoning.

2) The answer choices are accepted as given, even if there is _________ ______________, that is completely fine. Your job is to find the answer that evaluates the ___________ of the argument.

A

information; suspect; errors; new information; validity

How well did you know this?
1
Not at all
2
3
4
5
Perfectly
6
Q

Question Stem Examples:

“The answer to which one of the following questions would contribute most to an evaluation of the argument?”

“Clarification of which one of the following issues would be most important to an evaluation of the skeptics’ position?”

“Which one of the following would it be most helpful to know in order to judge whether what the scientist subsequently learned calls into question the hypothesis?”

A
How well did you know this?
1
Not at all
2
3
4
5
Perfectly
7
Q

Variance Test:

A

involves applying two polar opposite responses to an answer choice.

if each response yields a different effect on the conclusion of the argument, such that one response strengthens the conclusion of the argument, and the other weakens the conclusion, then it is the correct answer choice.

If each response does not yield a different effect on the conclusion of the argument, then it is not the correct answer choice.

How well did you know this?
1
Not at all
2
3
4
5
Perfectly
8
Q

Ex. of Variance Test:

Ex. Answer Choice: What percentage of people live near the nuclear plant?”

Start out with two polar opposite responses:
Test Response #1: 0%
Test Response #2: 100%

Evaluate each response’s effect on the conclusion. If one response strengthens the conclusion and the other weakens the conclusion, then the answer choice is correct.

A
How well did you know this?
1
Not at all
2
3
4
5
Perfectly
9
Q

Bear in Mind: The Variance Test is only supposed to be used once you’ve narrowed down your answers to two attractive contenders.

So, how do you narrow down your answer choices?

A

After having read the stimulus, and as you go through the answer choices, ask yourself, do any of these answer choices raise an issue that is irrelevant to the argument.

If you see that an answer choice raises an irrelevant question, eliminate it as a loser.

If you see that answer choice raises a relevant question, keep it as a possible contender.

How well did you know this?
1
Not at all
2
3
4
5
Perfectly
10
Q

Try an example problem:

  1. Advertisement: Most power head trimmers on the market do an adequate job of trimming hedges but many power hedge trimmers are dangerous to operate and can cause serious injury when used by untrained operators. Bolter Industries’ hedge trimmer has been tested by National Laboratories, the most trusted name in safety testing. So, you know, if you buy a Bolter’s, you’re buying a power hedge trimmer whose safety is assured.

The answer choice to which one of the following questions would be most useful in evaluating the truth of the conclusion drawn in the advertisement?

(A) Has National Laboratories performed safety tests
on other machines made by Bolter Industries?

(B) How important to the average buyer of a power
hedge trimmer is safety of operation?

(C) What were the results of National Laboratories’
tests of Bolter Industries’ hedge trimmer?

(D) Are there safer ways of trimming a hedge than
using a power hedge trimmer?

(E) Does any other power hedge trimmer on the
market do a better job of trimming hedges than
does Bolter Industries’ hedge trimmer?

A

First, Identify conclusion: “So, you know, if you buy a Bolter’s, you’re buying a power hedge trimmer whose safety is assured.”

(A) (Contender)
Test Response #1: Yes, NL has performed safety test on other machines made by BI.

Test Response #2: No, NL has not performed safety test on other machines made by BI.

(B) (Loser: introduces irrelevant question to conclusion of argument)

(C) (Contender; if the trimmers fail the safety test, it weakens the conclusion; if they pass, it strengthens the conclusion)
Test Response #1: Bolter Industries power hedge trimmers fail the NL’s safety test.

Test Response #2: Bolter Industries power hedge trimmers completely pass the NL’s safety test.

(D) (Loser: Introduces irrelevant question to conclusion of argument)

(E) (Loser: Introduces irrelevant question to conclusion of argument)

How well did you know this?
1
Not at all
2
3
4
5
Perfectly
11
Q

Pointer: If the question stem says something like, “ Which one of the following LEAST helps to evaluate the argument….” then you’re dealing with an _____________ question, where you need to identify the answer choice that DOES NOT help to evaluate the _______________ of the argument. This means the responses to your answer choice should not ___________ AND ____________; your responses shouldn’t have a different effect on the conclusion of the argument.

A

Except; validity; strengthen; weaken

How well did you know this?
1
Not at all
2
3
4
5
Perfectly
12
Q

Indicator Words and Phrases in Evaluate Questions:

A

“Evaluate”
“Judge”
“Assess”

How well did you know this?
1
Not at all
2
3
4
5
Perfectly
13
Q

–LOGICAL REASONING: Cannot Be True–

A
How well did you know this?
1
Not at all
2
3
4
5
Perfectly
14
Q

Cannot Be True

(Question Stem #, Family, Model, Definition)

A

Question Stem #: 13

Family: #4; Disprove
(Based on the information in the stimulus, we disprove the correct answer)

Model:
Stimulus

Answer Choice

Definition:
Question Stem asks you to choose the answer choice that cannot be true, or does not follow from the information in the stimulus (most weakened by the information in the stimulus)

How well did you know this?
1
Not at all
2
3
4
5
Perfectly
15
Q

In Cannot Be True Questions, the stimulus attacks ___ of ____ answer choices.

A

1; 5

How well did you know this?
1
Not at all
2
3
4
5
Perfectly
16
Q

Cannot Be True Questions can be viewed as…

1) Polar Opposite Must Be True Questions.

2) Reverse Weaken Questions

Explain each.

A

1) Polar Opposite Must Be True Questions
- Must be true questions: based on the information in the stimulus, we prove the correct answer choice.
- Cannot be true questions: based on the information in the stimulus, we disprove the correct answer choice (opposite function, same relationship)

2) Reverse Weaken Question
- Weaken Question (#8) Model (Family #3: Hurt):

Stimulus

Answer Choice

With Cannot Be True Questions, we simply REVERSE the arrow–reverse the relationship between stimulus and answer choices.

  • Cannot Be True Question (#12) Model (Family #4):

Stimulus

Answer Choice

17
Q

Test makers intersperse these Cannot Be True questions amongst the others in the logical reasoning set to __________ off test takers. These questions, if not trained properly, are very __________ consuming. You NEED to be able to eliminate __________ _____ _______ / ____________ answers, and choose __________ _____ _______ / ____________ answers.

A

throw; time; could be true; possible; cannot be true; impossible

18
Q

Rules that Apply to Cannot Be True Questions (3)

A

1) Accept the stimulus as given, and use the information in the stimulus to prove that answer choice cannot occur.

(you’ll see it in the wording of the question stem: “If all the claims made above are true…,” “If the statements above are true…”)

2) Answer choices that contain information that is not directly stated in the stimulus, or answer choices that combine separate items from the stimulus are could be true answers, and thus incorrect.

Answer choices that directly disagree with the information in the stimulus, are cannot be true answers, and thus correct.

3) Like Must Be True or Resolve the Paradox questions, the stimulus to a Cannot Be True question will often not have a conclusion

19
Q

Cannot Be True Question Stem Examples:

“If all of the claims made above are true, then each of the following could be true EXCEPT:”

“The argument can most reasonably be interpreted as an objection to which one of the following claims?”

“If the statements above are true, then which one of the following must be false.”

“The statements above, if true, most seriously undermine which one of the following assertions?”

“If the statements above are true and the archeologists hypothesis is correct, which one of the following CANNOT be true?”

“In which one of the following situations is the principle expressed most clearly violated?”

Identify the wording in each question stem that indicates it’s a Cannot Be True question:

A
  1. could be true EXCEPT
  2. objection
  3. Must Be False
  4. undermine
  5. CANNOT be true
  6. violated
20
Q

Ways in which Cannot Be True can be worded or expressed:

  1. Stating the answer choice cannot be true or does not follow.
  2. Stating that the answer could be true EXCEPT.
  3. Stating that an answer choice must be false.

Explain why each of these are ways in which Cannot Be True Questions can be expressed.

A
  1. Most self explanatory. If an answer choice cannot be true, it’s the correct answer, or if it doesn’t follow from the information in the stimulus, then it is the correct answer choice.
  2. Remember, could be true is the logical opposite of cannot be true. So if it’s an Except question to a could be true answer, then they’re asking for a cannot be true answer.
  3. Must be false is the logical equivalent of Cannot Be True.
21
Q

Example LSAT Question:

  1. Sharks have a higher ratio of cartilage mass to body mass than any other organism. They also have a greater resistance to cancer than any other organism. Shark cartilage contains a substance that inhibits tumor growth by stopping the development of a new blood network. In the past 20 years, none of the responses among terminal cancer patients to various therapeutic measures has become more positive than the response among those who consume shark cartilage.

If the claims made above are true, then each of the following could be true EXCEPT:

(A) No organism resists cancer better than sharks do,
but some resist cancer as well as sharks.

(B) The organism most susceptible to cancer has
a higher percentage of cartilage than some
organisms that are less susceptible to cancer.

(C) The substance in shark cartilage that inhibits
tumor growth is found in most organisms.

(D) In the past 20 years many terminal cancer
patients have improved dramatically following
many sorts of therapy

(E) Some organisms have immune systems more
efficient than a shark’s immune system.

A

Accept the information in the stimulus as given, and using that information choose an answer choice that cannot occur (eliminate all could be true/possible answers).

(A): Contender
(stimulus says shark cartilage resists cancer better than ANY OTHER organism, so it couldn’t be that there is an organism that resist cancer as well as a shark).

(B): Loser
(could be true that organisms most susceptible have higher percentage of cartilage in there body than some organisms that are less susceptible: it’s specifically shark cartilage that inhibits tumor growth; other cartilage of other types of organisms, could leave those organisms vulnerable to cancer, even if the organism itself is mostly made up of cartilage).

(C): Loser
(Could be true)

(D): Loser
(Could be true that many patients have improved due to many sorts of therapy, but that still allows the fact that no patient has had a more positive experience that shark cartilage consumption).

(E): Loser
(Could be true: introduces info that is not directly stated in the stimulus).

The correct answer choice is A.

22
Q

Two Notable Scenarios: 1) Numbers and Percentages, 2) Conditional Reasoning

  1. Numbers and Percentages
    - We know test makers are going to play upon some _______________ (7) that test takers have about numbers and percentages. However, in Cannot Be True question that deal with numbers and percentages as a stimulus scenario, the problem is going tell you enough _____________ for you to know what certain ____________ absolutely __________ occur. So, given that it’s a Cannot Be True question, any answer choice that says that these ____________ don’t occur is the correct answer.
  2. Conditional Reasoning:
    - Cannot Be True questions take place within several conditional reasoning stimulus scenarios, except for this one:
    when the ____________ condition occurs, but the ___________ condition doesn’t occur.

So, given that it’s a Cannot Be True question, the correct answer will follow that format.

A

misconceptions; information; outcomes; MUST; outcomes

sufficient; necessary